2003 HSC Mathematics Extension 2


\begin{Example}
(难度: \score{3}{5})解方程组
$$\begin{cases}
x+y+z=a, \\
x^2+y^2+z^2=b^2, \\
x^3+y^3+z^3=a^3.
\end{cases}$$
\end{Example}
\begin{proof}

\end{proof}

\begin{Example}
(难度: \score{3}{5})解方程组
$$\begin{cases}
x+y+z=a, \\
x^2+y^2+z^2=b, \\
x^3+y^3+z^3=c.
\end{cases}$$
\end{Example}
\begin{proof}

\end{proof}

 


$\sum_{n=0}^\infty\frac{(2n)!!}{(2n+1)!!}x^{2n+1}$ , x belongs to( 0, 1)

This is quite hard and I think it belongs to College Math. I will assume the (usual) convention that $0!!=1$.
Write $a_n:={{(2n)!!}\over {(2n+1)!!}}$ then $a_{n+1}= ({{2n+2}\over {2n+3}})a_n=a_n-{{a_n}\over {2n+3}}$.
Write $f(x)=\sum_0^\infty a_n x^{2n+1}$ then $f(x)=x+\sum_0^\infty (a_n-{{a_n}\over {2n+3}})x^{2n+3}=x+x^2 f(x)-\sum_0^\infty {{a_n x^{2n+3}}\over {2n+3}}$.
Differentiating we find $f'(x)=1+2xf(x)+x^2 f'(x)-xf(x)$, hence $(1-x^2)f'(x)-xf(x)=1$.
The reduced equation has solution $f={c\over {\sqrt{1-x^2}}}$ hence we try $f={g\over {\sqrt{1-x^2}}}$ by Variation of Constants.
Then we get $g'\sqrt{1-x^2}=1 \implies g=\arcsin x\implies f={{\arcsin x+c}\over {\sqrt{1-x^2}}}$.
Since $f$ is an odd function we have $c=0$.


\begin{example}
1
\end{example}
\begin{proof}

\end{proof}


\begin{example}
$1\sim n$这$n$个正整数可被均分为两组,使得两组数之和及它们的平方和均相等.求出所有满足条件的正整数$n$.
\end{example}
\begin{proof}
首先由均分可知$n$为偶数,且每组的和均为$\frac{1}{2}(1+2+\cdots+n)=\frac{n(n+1)}{4}$,是一个正整数.由$n$为偶数和$n+1$为奇数可知, $4$整除$n$,故$n=4k$ ($k$为正整数).显然$n=4$不符合题意.

%而$\frac{1}{2}(1^2+2^2+\cdots+n^2)=\frac{n(n+1)(2n+1)}{12}$为正整数.

下面证明,当$n=4k$ ($k\geqslant 2$且$k$为正整数)时,满足题意.

引理1:连续8个自然数可分两组,每组和及平方和相等.

证明:注意到
\begin{align*}
&n+\left( n+3 \right) +\left( n+5 \right) +\left( n+6 \right) =4n+14
\\
&=\left( n+1 \right) +\left( n+2 \right) +\left( n+4 \right) +\left( n+7 \right)
\end{align*}

\begin{align*}
&\left( n+1 \right) ^2+\left( n+2 \right) ^2+\left( n+4 \right) ^2+\left( n+7 \right) ^2
\\
&-\left[ n^2+\left( n+3 \right) ^2+\left( n+5 \right) ^2+\left( n+6 \right) ^2 \right]
\\
&=\left[ \left( n+7 \right) ^2-\left( n+6 \right) ^2 \right] -\left[ \left( n+5 \right) ^2-\left( n+4 \right) ^2 \right]
\\
&-\left[ \left( n+3 \right) ^2-\left( n+2 \right) ^2 \right] +\left[ \left( n+1 \right) ^2-n^2 \right]
\\
&=\left( 2n+13 \right) -\left( 2n+9 \right) -\left( 2n+5 \right) +\left( 2n+1 \right) =0,
\end{align*}
则两组数: $n,n+3,n+5,n+6$和$n+1,n+2,n+4,n+7$的和与平方和都相等.

对于$n=8k$ ($k$为正整数),可将$1\sim n$这$n$个正整数每八个均分为两组:

a) $1,4,6,7;9,12,14,15;\cdots;8k-7,8k-4,8k-2,8k-1$

b) $2,3,5,8;10,11,13,16;\cdots;8k-6,8k-5,8k-3,8k$

因此所有的$n=8k$ ($k$为正整数)满足题意.


引理2:连续12个自然数可分两组,每组和及平方和相等.

证明:注意到
\begin{align*}
&1+3+7+8+9+11=39
\\
&=2+4+5+6+10+12.
\end{align*}
以及
\begin{align*}
&1^2+3^2+7^2+8^2+9^2+11^2=325
\\
&=2^2+4^2+5^2+6^2+10^2+12^2
\end{align*}
可知
\begin{align*}
&\left( n+1 \right) +\left( n+3 \right) +\left( n+7 \right) +\left( n+8 \right) +\left( n+9 \right) +\left( n+11 \right)
\\
&=\left( n+2 \right) +\left( n+4 \right) +\left( n+5 \right) +\left( n+6 \right) +\left( n+10 \right) +\left( n+12 \right)
\end{align*}

\begin{align*}
&\left( n+1 \right) ^2+\left( n+3 \right) ^2+\left( n+7 \right) ^2+\left( n+8 \right) ^2+\left( n+9 \right) ^2+\left( n+11 \right) ^2
\\
&=\left( n+2 \right) ^2+\left( n+4 \right) ^2+\left( n+5 \right) ^2+\left( n+6 \right) ^2+\left( n+10 \right) ^2+\left( n+12 \right) ^2.
\end{align*}

则两组数: $n+1,n+3,n+7,n+8,n+9,n+11$和$n+2,n+4,n+5,n+6,n+10,n+12$的和与平方和都相等.

对于$n=8k+4$ ($k$为正整数),对于$1\sim n$这$n$个正整数,可以先将前12个数均分为两组,接着每八个均分为两组:

a) $1,3,7,8,9,11;13,16,18,19;\cdots;8k-3,8k,8k+2,8k+3$

b) $2,4,5,6,10,12;14,15,17,20;\cdots;8k-2,8k-1,8k+1,8k+4$

因此所有的$n=8k+4$ ($k$为正整数)满足题意.

综上所述, 只有当$n=4k$ ($k\geqslant 2$且$k$为正整数)时,满足题意.
\end{proof}

 


\begin{example}
设$f(x)=ax^2+bx+c$, $a,b,c$为实数,如果对于所有适合$-1\leqslant x \leqslant 1$的$x$值,都有$-1\leqslant f(x) \leqslant 1$成立,则对这些$x$的值有$-4\leqslant 2ax+b\leqslant 4$. (匈牙利, 1914)
\end{example}
\begin{proof}

$$
\begin{cases}
f\left( -1 \right) =a-b+c\\
f\left( 0 \right) =c\\
f\left( 1 \right) =a+b+c
\end{cases}$$
可知
$$
\begin{cases}
a=\frac{f\left( 1 \right) +f\left( -1 \right)}{2}-f\left( 0 \right)\\
b=\frac{f\left( 1 \right) -f\left( -1 \right)}{2}\\
c=f\left( 0 \right).
\end{cases}$$

$$
g\left( x \right) =2ax+b=\left[ f\left( 1 \right) +f\left( -1 \right) -2f\left( 0 \right) \right] x+\frac{f\left( 1 \right) -f\left( -1 \right)}{2}
$$
为一次函数,最值在端点处取得.


\begin{align*}
-4 &\leqslant g\left( 1 \right) =\frac{3}{2}f\left( 1 \right) +\frac{1}{2}f\left( -1 \right) -2f\left( 0 \right) \leqslant 4,
\\
-4 &\leqslant g\left( -1 \right) =-\frac{1}{2}f\left( 1 \right) -\frac{3}{2}f\left( -1 \right) +2f\left( 0 \right) \leqslant 4
\end{align*}
可知对$-1\leqslant x \leqslant 1$,都有$-1\leqslant f(x) \leqslant 1$成立,则对这些$x$的值有$-4\leqslant 2ax+b\leqslant 4$.


\end{proof}

此题的背景是契比雪夫多项式的马尔科夫定理:如果具有实系数的$n$次多项式
$$f(x)=a_0+a_1x+a_2x^2+\cdots +a_nx^n$$
对所有的$-1\leqslant x \leqslant 1$满足不等式$-1\leqslant f(x) \leqslant 1$.
那么它的导函数满足不等式
$-n^2\leqslant f'(x)\leqslant n^2$.

虽然背景是高等的,但解法只用到一次函数$g(x)=2ax+b$的单调性、取值的技巧和不等式的放缩运算.


\begin{Example}
(2007年台湾数学能力竞赛决赛,难度: \score{3}{5})有一正整数列$1,2,3,\cdots,2n-1,2n$,现从中挑出$n$个数,从大到小排列依次为$a_1,a_2,\cdots,a_n$,另$n$个数从小到大排列依次为$b_1,b_2,\cdots,b_n$.
求$|a_1-b_1|+|a_2-b_2|+\cdots+|a_n-b_n|$之所有可能的值.
\end{Example}
\begin{proof}
令$n+1,n+2,n+3,\cdots,2n$为大数, $1,2,3,\cdots,n$为小数.

设$a_i$中必也有$n-k$个小数,则$b_i$中必有$n-k$个大数, $k$个小数,其中$i=1,2,3,\cdots,n,0\leqslant k\leqslant n,k\in \mathbb{Z}$.

令$a_1,a_2,\cdots,a_k,b_{k+1},b_{k+2},\cdots,b_n$为大数, $b_1,b_2,\cdots,b_k,a_{k+1},a _{k+2},\cdots,a_n$为小数.


\begin{align*}
&\left| a_1-b_1 \right|+\left| a_2-b_2 \right|+\cdots +\left| a_n-b_n \right|\\
&=\left| a_1-b_1 \right|+\left| a_2-b_2 \right|+\cdots +\left| a_k-b_k \right|+\left| a_{k+1}-b_{k+1} \right|+\left| a_{k+2}-b_{k+2} \right|+\cdots +\left| a_n-b_n \right|\\
&=\left( a_1-b_1 \right) +\left( a_2-b_2 \right) +\cdots +\left( a_k-b_k \right) +\left( b_{k+1}-a_{k+1} \right) +\left( b_{k+2}-a_{k+2} \right) +\cdots +\left( b_n-a_n \right)\\
&=\left[ \left( n+1 \right) +\left( n+2 \right) +\cdots +\left( 2n \right) \right] -\left( 1+2+\cdots +n \right) =n^2.
\end{align*}
\end{proof}

\begin{Example}
(难度: \score{3}{5})问题求解:

一、连续8个自然数可分两组,每组和及平方和相等.

1) $n,n+3,n+5,n+6$

2) $n+1,n+2,n+4,n+7$

两组平方和都为: $4n^2+28n+70$.


二、类似地,连续$2^{m+1}$个自然数可分两组,每组的和、平方和、立方和, $\cdots$,直到$m$次方和均相等,并找出分组的方法.

三、推广:连续$d^{m+1}$个自然数,其中$d\geqslant 2$,可分$d$组,每组的和、平方和、立方和, $\cdots$,直到$m$次方和均相等.并找出分组的方法.
\end{Example}
\begin{proof}

\end{proof}

 

2016, 2015, 2014, 2010, 2009, 2008, 2006, 2004, 2002, 2001

\begin{example}
(2001年HSC) (a) (i)证明对所有实数$a$和$b$,有$2ab\leqslant a^2+b^2$.

由此推出对所有实数$a,b$和$c$,有$3(ab+bc+ca)\leqslant (a+b+c)^2$.

(ii)假设$a,b$和$c$为三角形的三边长.解释为什么$(b-c)^2\leq a^2$.推导出$(a+b+c)^2\leq 4(ab+bc+ca)$.

(b) (i)解释为什么,对于$\alpha>0$,有
$$\int_{0}^{1}x^\alpha e^xdx<\frac{3}{\alpha+1}.$$
(设$e<3$.)

(ii)由归纳法说明,对于$n=0,1,2,\cdots$存在整数$a_n$和$b_n$使得
$$\int_{0}^{1}x^ne^xdx=a_n+b_ne.$$


(iii)假设$r$为正有理数,使得$r=\frac{p}{q}$,其中$p$和$q$是正整数.说明,对于所有整数$a$和$b$,要么$|a+br|=0$,要么$|a+br|\geqslant \frac{1}{q}$.

(iv)证明$e$是无理数.
\end{example}
\begin{proof}

\end{proof}

\begin{example}
(2002年HSC) (a)设$m$为正整数.

(i)由De Moivre定理,证明
\begin{align*}
\sin (2m+1)\theta=\binom{2m+1}{1}\cos^{2m}\theta\sin\theta
-\binom{2m+1}{3}\cos^{2m-2}\theta\sin^3\theta\\
+\cdots+(-1)^m\sin^{2m+1}\theta.
\end{align*}

(ii)推导出多项式
$$p(x)=\binom{2m+1}{1}x^{m}-\binom{2m+1}{3}x^{m-1}
+\cdots+(-1)^m$$
有$m$个不同的根
$$\alpha_k=\cot^2\left(\frac{k\pi}{2m+1}\right)\quad \text{其中}k=1,2,\cdots,m.$$


(iii)证明
$$\cot^2\left(\frac{\pi}{2m+1}\right)
+\cot^2\left(\frac{2\pi}{2m+1}\right)+\cdots+
\cot^2\left(\frac{m\pi}{2m+1}\right)=\frac{m(2m-1)}{3}.$$


(iv)对于$0<\theta<\frac{\pi}{2}$,有$\cot\theta<\frac{1}{\theta}$.

推导出
$$
\frac{\pi ^2}{6}<\left( \frac{1}{1^2}+\frac{1}{2^2}+\cdots +\frac{1}{m^2} \right) \frac{\left( 2m+1 \right) ^2}{2m\left( 2m-1 \right)}.
$$
\end{example}
\begin{proof}

\end{proof}

 

\begin{example}
(2004年HSC)对于$n=0,1,2,\cdots$,设$I_n=\int_{0}^{\frac{\pi}{4}}\tan^nxdx$
和$J_n=(-1)^nI_{2n}$.

(i)说明$I_n+I_{n-2}=\frac{1}{n+1}$.

(ii)对于$n\geqslant 1$,推出$J_n-J_{n-1}=\frac{(-1)^n}{2n-1}$.

(iii)证明$J_m=\frac{\pi}{4}+\sum_{n=1}^{m}\frac{(-1)^n}{2n-1}$.

(iv)利用换元$u=\tan x$说明$I_n=\int_{0}^{1}\frac{u^n}{1+u^2}du$.

(v)推出$0\leqslant I_n\leqslant\frac{1}{n+1}$,并证明当$n\to \infty$时, $J_n\to 0$.
\end{example}
\begin{proof}

\end{proof}

%https://terrytao.wordpress.com/2016/10/23/another-problem-about-power-series/
%https://terrytao.wordpress.com/2016/09/18/246a-notes-0-the-complex-numbers/
%https://terrytao.wordpress.com/2016/09/22/246a-notes-1-complex-differentiation/
%UCLA博士资格,https://secure.math.ucla.edu/gradquals/hbquals.php

\begin{example}
(2010年HSC)令
$$A_n=\int_{0}^{\frac{\pi}{2}}\cos^{2n}xdx\quad \text{和}\quad B_n=\int_{0}^{\frac{\pi}{2}}x^2\cos^{2n}xdx,$$
其中$n$是整数, $n\geqslant 0$. (注意到$A_n>0,B_n>0$.)

(a)说明$nA_n=\frac{2n-1}{2}A_{n-1},n\geqslant 1$.

(b)利用$A_n$的分部积分,说明
$$A_n=2n\int_{0}^{\frac{\pi}{2}}x\sin x\cos^{2n-1}xdx,\quad n\geqslant 1.$$

(c)对(b)中的积分进行分部积分,说明
$$\frac{A_n}{n^2}=\frac{(2n-1)}{n}B_{n-1}-2B_n,\quad n\geqslant 1.$$

(d)利用(a)和(c)证明
$$
\frac{1}{n^2}=2\left( \frac{B_{n-1}}{A_{n-1}}-\frac{B_n}{A_n} \right),\quad n\geqslant 1.
$$

(e)证明$\sum_{k=1}^{n}\frac{1}{k^2}=\frac{\pi^2}{6}-2\frac{B_n}{A_n}$.

(f)对于$0\leqslant x\leqslant \frac{\pi}{2}$,利用不等式$\sin x\geqslant \frac{2}{\pi}x$证明
$$
B_n\leqslant \int_0^{\frac{\pi}{2}}{x^2\left( 1-\frac{4x^2}{\pi ^2} \right) ^ndx}.
$$

(g)证明
$$
\int_0^{\frac{\pi}{2}}{x^2\left( 1-\frac{4x^2}{\pi ^2} \right) ^ndx}=\frac{\pi ^2}{8\left( n+1 \right)}\int_0^{\frac{\pi}{2}}{\left( 1-\frac{4x^2}{\pi ^2} \right) ^{n+1}dx}.
$$

(h)从(f)到(g)可得
$$
B_n\leqslant \frac{\pi ^2}{8\left( n+1 \right)}\int_0^{\frac{\pi}{2}}{\left( 1-\frac{4x^2}{\pi ^2} \right) ^{n+1}dx}.
$$
在此不等式中利用换元$x=\frac{\pi}{2}\sin t$证明
$$
B_n\leqslant \frac{\pi ^3}{16\left( n+1 \right)}\int_0^{\frac{\pi}{2}}{\cos ^{2n+3}tdt}<\frac{\pi ^3}{16\left( n+1 \right)}A_n.
$$

(i)利用(e)推出
$$
\frac{\pi ^2}{6}-\frac{\pi ^3}{8\left( n+1 \right)}\leqslant \sum_{k=1}^n{\frac{1}{k^2}}<\frac{\pi ^2}{6}.
$$


(j)求$\lim_{n\rightarrow \infty} \sum_{k=1}^n{\frac{1}{k^2}}$.
\end{example}
\begin{proof}

\end{proof}

\begin{example}
(2001年HSC)
\end{example}
\begin{proof}

\end{proof}

\begin{example}
(2001年HSC)
\end{example}
\begin{proof}

\end{proof}

 


HSC Mathematics Extension 2

Higher School Certificate,简称HSC,是一个证书授予那些在新南威尔士州中完成高中学习(11和12年级)并通过考试的高中学生。类似于中国的高考和高中毕业证书。HSC是从1967年引进新南威尔士州使用,最后主要修订是在2001年。HSC当前是新南威尔士州的教育部(Board of Studies of NSW引)管理和发展。

总分120分

尝试问题1-8

所有问题都有同等分值,在单独的小册子中回答每个问题.可提供额外的小册子.

问题1 (15分)使用单独的小册子写解答.\hfill \textbf{分值}


(a)求$\int_0^1{\frac{e^x}{\left( 1+e^x \right) ^2}dx}$. \hfill \textbf{2}

(b)利用分部积分求 \hfill \textbf{3}
$$\int x^3\log_exdx.$$

(c)利用配方和标准积分表,求\hfill \textbf{2}
$$\int\frac{dx}{\sqrt{x^2-2x+5}}.$$


(d) (i)求实数$a$和$b$,满足\hfill \textbf{2}
$$
\frac{5x^2-3x+13}{\left( x-1 \right) \left( x^2+4 \right)}\equiv \frac{a}{x-1}+\frac{bx-1}{x^2+4}.
$$

(ii)求$\int_0^1{\frac{5x^2-3x+13}{\left( x-1 \right) \left( x^2+4 \right)}dx}$. \hfill \textbf{2}

(e)利用换元$x=3\sin\theta$计算 \hfill \textbf{4}
$$
\int_0^{\frac{3}{\sqrt{2}}}{\frac{dx}{\left( 9-x^2 \right) ^{\frac{3}{2}}}}.
$$


问题2 (15分)使用单独的小册子写解答.\hfill \textbf{分值}

(a)设$z=2+i$和$w=1-i$.

将下列各数的值写成$x+iy$的形式,

(i) $z\overline{w}$; \hfill \textbf{1}


(ii) $\frac{4}{z}$. \hfill \textbf{1}


(b)设$\alpha=-1+i$.

(i)将$\alpha$表示成模—辐角的形式($z=|z|(\cos\theta+i\sin\theta)$). \hfill \textbf{2}

(ii)说明$\alpha$是方程$z^4+4=0$的根. \hfill \textbf{1}


(iii)由此,进一步找出多项式$z^4+4$的实二次因式. \hfill \textbf{2}


(c)在复平面上绘制不等式
$$|z-1-i|<2\qquad \text{和}\qquad 0<\arg (z-1-i)<\frac{\pi}{4}$$
同时成立的区域. \hfill \textbf{3}

(d)由棣莫弗定理(de Moivre’s theorem)和$(\cos\theta+i\sin\theta)^5$的展开式,将$\cos 5\theta$表示成$\cos\theta$的多项式. \hfill \textbf{3}

(e)设$z$位于单位圆上,且$0\leqslant \arg(z)\leqslant \frac{\pi}{2}$.证明$2\arg(z+1)=\arg(z)$. \hfill \textbf{2}

问题3 (15分)使用单独的小册子写解答.\hfill \textbf{分值}

(a)图中显示了$y=f(x)$的图像.

分别绘制出如下函数的草图:

(i) $y=\frac{1}{f(x)}$ \hfill \textbf{2}

(ii) $y=f(x)+|f(x)|$ \hfill \textbf{2}

(iii) $y=(f(x))^2$ \hfill \textbf{1}

(iv) $y=e^{f(x)}$. \hfill \textbf{2}

(b)求椭圆$\frac{x^2}{9}+\frac{y^2}{4}=1$的离心率、焦点和准线(directrices)方程. \hfill \textbf{3}

(c)由曲线$y=(x-1)(3-x)$和$x$轴围成的区域绕直线$x=3$旋转形成一个立体.当该区域旋转时,高度为$y$的水平线段$\ell$扫出一个环形区域.


(i)说明高度为$y$的该环形区域的面积为$4\pi\sqrt{1-y}$. \hfill \textbf{3}

(ii)求该立体的体积. \hfill \textbf{2}


问题4 (15分)使用单独的小册子写解答.\hfill \textbf{分值}


(a)质量为$m$的粒子$P$在半径为$r$的圆上以恒定角速度$\omega$运动,其在时间$t$时的位置由下式给出:
\begin{align*}
x&=r\cos\theta,\\
y&=r\sin\theta,
\end{align*}
其中$\theta=\omega t$.

(i)说明作用在$P$上有一个大小为$mr\omega^2$的向心力. \hfill \textbf{3}

(ii)一颗质量为$m$的通信卫星距离地球中心$r$,以恒定角速度$\omega$绕地球运行.地球对卫星的引力是$\frac{Am}{r^2}$,其中$A$是常数.考虑到卫星上的所有其他力都可以忽略不计,说明 \hfill \textbf{1}
$$r=\sqrt[3]{\frac{A}{\omega^2}}.$$


(b) (i)推导双曲线$\frac{x^2}{a^2}-\frac{y^2}{b^2}=1$在点$P(a\sec\theta,b\tan\theta)$处的切线方程. \hfill \textbf{2}

(ii)证明该切线与双曲线的渐近线交于点 \hfill \textbf{2}
$$
A\left( \frac{a\cos \theta}{1-\sin \theta},\frac{b\cos \theta}{1-\sin \theta} \right) \qquad \text{和}\qquad B\left( \frac{a\cos \theta}{1+\sin \theta},\frac{-b\cos \theta}{1+\sin \theta} \right).
$$

(iii)证明三角形$OAB$的面积为$ab$. \hfill \textbf{4}

(c) 大厅有$n$扇门.假设有$n$个人,每个人随机选择任意一扇门进入大厅.

(i)共有多少种方法可以做到这一点? \hfill \textbf{1}

(ii)至少有一扇门不会被任何人选中的概率有多大? \hfill \textbf{2}

问题5 (15分)使用单独的小册子写解答.\hfill \textbf{分值}

(a)设$\alpha,\beta$和$\gamma$为$x^3+px+q=0$的三根,定义$s_n$为
$$s_n=\alpha^n+\beta^n+\gamma^n,\qquad \text{对于}n=1,2,3,\cdots$$

(i)解释$s_1=0$的原因,说明$s_2=-2p$且$s_3=-3q$. \hfill \textbf{3}

(ii)证明对于$n>3$有 \hfill \textbf{2}
$$s_n=-ps_{n-2}-qs_{n-3}.$$

(iii)推断出 \hfill \textbf{2}
$$
\frac{\alpha ^5+\beta ^5+\gamma ^5}{5}=\left( \frac{\alpha ^2+\beta ^2+\gamma ^2}{2} \right) \left( \frac{\alpha ^3+\beta ^3+\gamma ^3}{3} \right).
$$

(b) 一个质量为$m$的粒子从一座非常高的建筑物的顶部$O$抛出,初始速度为$u$,与水平面成角$\alpha$.粒子受到重力,并且在水平方向和垂直方向上都受到与其速度成正比的阻力.水平和垂直方向上的运动方程分别为
$$\ddot{x}=-k\dot{x}\qquad \text{和} \qquad \ddot{x}=-k\dot{y}-g,$$
式中$k$为常数,重力加速度为$g$ (无需说明这些)

(i)从有关的运动方程中导出结果$\dot{x}=ue^{-kt}\cos\alpha$. \hfill \textbf{2}

(ii)验证$\dot{y}=\frac{1}{k}\left( \left( ku\sin \alpha +g \right) e^{-kt}-g \right)$是否满足适当的运动方程和初始条件. \hfill \textbf{2}

(iii)当粒子达到最大高度时,求$t$的值. \hfill \textbf{2}


(iv)粒子水平位移的极限值是多少? \hfill \textbf{2}

posted on 2020-09-08 19:19  Eufisky  阅读(258)  评论(0编辑  收藏  举报

导航